What’s a Hypothesis Space?

Last updated: March 18, 2024

what is hypothesis space in machine learning

  • Math and Logic

announcement - icon

Baeldung Pro comes with both absolutely No-Ads as well as finally with Dark Mode , for a clean learning experience:

>> Explore a clean Baeldung

Once the early-adopter seats are all used, the price will go up and stay at $33/year.

1. Introduction

Machine-learning algorithms come with implicit or explicit assumptions about the actual patterns in the data. Mathematically, this means that each algorithm can learn a specific family of models, and that family goes by the name of the hypothesis space.

In this tutorial, we’ll talk about hypothesis spaces and how to choose the right one for the data at hand.

2. Hypothesis Spaces

Let’s say that we have a binary classification task and that the data are two-dimensional. Our goal is to find a model that classifies objects as positive or negative. Applying Logistic Regression , we can get the models of the form:

which estimate the probability that the object at hand is positive.

2.1. Hypotheses and Assumptions

The underlying assumption of hypotheses ( 1 ) is that the boundary separating the positive from negative objects is a straight line. So, every hypothesis from this space corresponds to a straight line in a 2D plane. For instance:

Two Classification Hypotheses

2.2. Regression

3. expressivity of a hypothesis space.

We could informally say that one hypothesis space is more expressive than another if its hypotheses are more diverse and complex.

We may underfit the data if our algorithm’s hypothesis space isn’t expressive enough. For instance, linear hypotheses aren’t particularly good options if the actual data are extremely non-linear:

Non-linear Data

So, training an algorithm that has a very expressive space increases the chance of completely capturing the patterns in the data. However, it also increases the risk of overfitting. For instance, a space containing the hypotheses of the form:

would start modelling the noise, which we see from its decision boundary:

A too complex hypothesis

Such models would generalize poorly to unseen data.

3.1. Expressivity vs. Interpretability

Additionally, even if a complex hypothesis has a good generalization capability, it may be unusable in practice because it’s too complicated to understand or compute. What’s more, intricated hypotheses offer limited insight into the real-world process that generated the data. For example, a quadratic model:

4. How to Choose the Hypothesis Space?

We need to find the right balance between expressivity and simplicity. Unfortunately, that’s easier said than done. Most of the time, we need to rely on our intuition about the data.

So, we should start by exploring the dataset, using visualizations as much as possible. For instance, we can conclude that a straight line isn’t likely to be an adequate boundary for the above classification data. However, a high-order curve would probably be too complex even though it might split the dataset into two classes without an error.

A second-degree curve might be the compromise we seek, but we aren’t sure. So, we start with the space of quadratic hypotheses:

We get a model whose decision boundary appears to be a good fit even though it misclassifies some objects:

An adequate hypothesis

Since we’re satisfied with the model, we can stop here. If that hadn’t been the case, we could have tried a space of cubic models. The idea would be to iteratively try incrementally complex families until finding a model that both performs well and is easy to understand.

4. Conclusion

In this article, we talked about hypotheses spaces in machine learning. An algorithm’s hypothesis space contains all the models it can learn from any dataset.

The algorithms with too expressive spaces can generalize poorly to unseen data and be too complex to understand, whereas those with overly simple hypotheses may underfit the data. So, when applying machine-learning algorithms in practice, we need to find the right balance between expressivity and simplicity.

home

Machine Learning

  • Machine Learning Tutorial
  • Machine Learning Applications
  • Life cycle of Machine Learning
  • Install Anaconda & Python
  • AI vs Machine Learning
  • How to Get Datasets
  • Data Preprocessing
  • Supervised Machine Learning
  • Unsupervised Machine Learning
  • Supervised vs Unsupervised Learning

Supervised Learning

  • Regression Analysis
  • Linear Regression
  • Simple Linear Regression
  • Multiple Linear Regression
  • Backward Elimination
  • Polynomial Regression

Classification

  • Classification Algorithm
  • Logistic Regression
  • K-NN Algorithm
  • Support Vector Machine Algorithm
  • Na�ve Bayes Classifier

Miscellaneous

  • Classification vs Regression
  • Linear Regression vs Logistic Regression
  • Decision Tree Classification Algorithm
  • Random Forest Algorithm
  • Clustering in Machine Learning
  • Hierarchical Clustering in Machine Learning
  • K-Means Clustering Algorithm
  • Apriori Algorithm in Machine Learning
  • Association Rule Learning
  • Confusion Matrix
  • Cross-Validation
  • Data Science vs Machine Learning
  • Machine Learning vs Deep Learning
  • Dimensionality Reduction Technique
  • Machine Learning Algorithms
  • Overfitting & Underfitting
  • Principal Component Analysis
  • What is P-Value
  • Regularization in Machine Learning
  • Examples of Machine Learning
  • Semi-Supervised Learning
  • Essential Mathematics for Machine Learning
  • Overfitting in Machine Learning
  • Types of Encoding Techniques
  • Feature Selection Techniques in Machine Learning
  • Bias and Variance in Machine Learning
  • Machine Learning Tools
  • Prerequisites for Machine Learning
  • Gradient Descent in Machine Learning
  • Machine Learning Experts Salary in India
  • Machine Learning Models
  • Machine Learning Books
  • Linear Algebra for Machine learning
  • Types of Machine Learning
  • Feature Engineering for Machine Learning
  • Top 10 Machine Learning Courses in 2021
  • Epoch in Machine Learning
  • Machine Learning with Anomaly Detection
  • What is Epoch
  • Cost Function in Machine Learning
  • Bayes Theorem in Machine learning
  • Perceptron in Machine Learning
  • Entropy in Machine Learning
  • Issues in Machine Learning
  • Precision and Recall in Machine Learning
  • Genetic Algorithm in Machine Learning
  • Normalization in Machine Learning
  • Adversarial Machine Learning
  • Basic Concepts in Machine Learning
  • Machine Learning Techniques
  • Demystifying Machine Learning
  • Challenges of Machine Learning
  • Model Parameter vs Hyperparameter
  • Hyperparameters in Machine Learning
  • Importance of Machine Learning
  • Machine Learning and Cloud Computing
  • Anti-Money Laundering using Machine Learning
  • Data Science Vs. Machine Learning Vs. Big Data
  • Popular Machine Learning Platforms
  • Deep learning vs. Machine learning vs. Artificial Intelligence
  • Machine Learning Application in Defense/Military
  • Machine Learning Applications in Media
  • How can Machine Learning be used with Blockchain
  • Prerequisites to Learn Artificial Intelligence and Machine Learning
  • List of Machine Learning Companies in India
  • Mathematics Courses for Machine Learning
  • Probability and Statistics Books for Machine Learning
  • Risks of Machine Learning
  • Best Laptops for Machine Learning
  • Machine Learning in Finance
  • Lead Generation using Machine Learning
  • Machine Learning and Data Science Certification
  • What is Big Data and Machine Learning
  • How to Save a Machine Learning Model
  • Machine Learning Model with Teachable Machine
  • Data Structure for Machine Learning
  • Hypothesis in Machine Learning
  • Gaussian Discriminant Analysis
  • How Machine Learning is used by Famous Companies
  • Introduction to Transfer Learning in ML
  • LDA in Machine Learning
  • Stacking in Machine Learning
  • CNB Algorithm
  • Deploy a Machine Learning Model using Streamlit Library
  • Different Types of Methods for Clustering Algorithms in ML
  • EM Algorithm in Machine Learning
  • Machine Learning Pipeline
  • Exploitation and Exploration in Machine Learning
  • Machine Learning for Trading
  • Data Augmentation: A Tactic to Improve the Performance of ML
  • Difference Between Coding in Data Science and Machine Learning
  • Data Labelling in Machine Learning
  • Impact of Deep Learning on Personalization
  • Major Business Applications of Convolutional Neural Network
  • Mini Batch K-means clustering algorithm
  • What is Multilevel Modelling
  • GBM in Machine Learning
  • Back Propagation through time - RNN
  • Data Preparation in Machine Learning
  • Predictive Maintenance Using Machine Learning
  • NLP Analysis of Restaurant Reviews
  • What are LSTM Networks
  • Performance Metrics in Machine Learning
  • Optimization using Hopfield Network
  • Data Leakage in Machine Learning
  • Generative Adversarial Network
  • Machine Learning for Data Management
  • Tensor Processing Units
  • Train and Test datasets in Machine Learning
  • How to Start with Machine Learning
  • AUC-ROC Curve in Machine Learning
  • Targeted Advertising using Machine Learning
  • Top 10 Machine Learning Projects for Beginners using Python
  • What is Human-in-the-Loop Machine Learning
  • What is MLOps
  • K-Medoids clustering-Theoretical Explanation
  • Machine Learning Or Software Development: Which is Better
  • How does Machine Learning Work
  • How to learn Machine Learning from Scratch
  • Is Machine Learning Hard
  • Face Recognition in Machine Learning
  • Product Recommendation Machine Learning
  • Designing a Learning System in Machine Learning
  • Recommendation System - Machine Learning
  • Customer Segmentation Using Machine Learning
  • Detecting Phishing Websites using Machine Learning
  • Hidden Markov Model in Machine Learning
  • Sales Prediction Using Machine Learning
  • Crop Yield Prediction Using Machine Learning
  • Data Visualization in Machine Learning
  • ELM in Machine Learning
  • Probabilistic Model in Machine Learning
  • Survival Analysis Using Machine Learning
  • Traffic Prediction Using Machine Learning
  • t-SNE in Machine Learning
  • BERT Language Model
  • Federated Learning in Machine Learning
  • Deep Parametric Continuous Convolutional Neural Network
  • Depth-wise Separable Convolutional Neural Networks
  • Need for Data Structures and Algorithms for Deep Learning and Machine Learning
  • Geometric Model in Machine Learning
  • Machine Learning Prediction
  • Scalable Machine Learning
  • Credit Score Prediction using Machine Learning
  • Extrapolation in Machine Learning
  • Image Forgery Detection Using Machine Learning
  • Insurance Fraud Detection -Machine Learning
  • NPS in Machine Learning
  • Sequence Classification- Machine Learning
  • EfficientNet: A Breakthrough in Machine Learning Model Architecture
  • focl algorithm in Machine Learning
  • Gini Index in Machine Learning
  • Rainfall Prediction using ML
  • Major Kernel Functions in Support Vector Machine
  • Bagging Machine Learning
  • BERT Applications
  • Xtreme: MultiLingual Neural Network
  • History of Machine Learning
  • Multimodal Transformer Models
  • Pruning in Machine Learning
  • ResNet: Residual Network
  • Gold Price Prediction using Machine Learning
  • Dog Breed Classification using Transfer Learning
  • Cataract Detection Using Machine Learning
  • Placement Prediction Using Machine Learning
  • Stock Market prediction using Machine Learning
  • How to Check the Accuracy of your Machine Learning Model
  • Interpretability and Explainability: Transformer Models
  • Pattern Recognition in Machine Learning
  • Zillow Home Value (Zestimate) Prediction in ML
  • Fake News Detection Using Machine Learning
  • Genetic Programming VS Machine Learning
  • IPL Prediction Using Machine Learning
  • Document Classification Using Machine Learning
  • Heart Disease Prediction Using Machine Learning
  • OCR with Machine Learning
  • Air Pollution Prediction Using Machine Learning
  • Customer Churn Prediction Using Machine Learning
  • Earthquake Prediction Using Machine Learning
  • Factor Analysis in Machine Learning
  • Locally Weighted Linear Regression
  • Machine Learning in Restaurant Industry
  • Machine Learning Methods for Data-Driven Turbulence Modeling
  • Predicting Student Dropout Using Machine Learning
  • Image Processing Using Machine Learning
  • Machine Learning in Banking
  • Machine Learning in Education
  • Machine Learning in Healthcare
  • Machine Learning in Robotics
  • Cloud Computing for Machine Learning and Cognitive Applications
  • Credit Card Approval Using Machine Learning
  • Liver Disease Prediction Using Machine Learning
  • Majority Voting Algorithm in Machine Learning
  • Data Augmentation in Machine Learning
  • Decision Tree Classifier in Machine Learning
  • Machine Learning in Design
  • Digit Recognition Using Machine Learning
  • Electricity Consumption Prediction Using Machine Learning
  • Data Analytics vs. Machine Learning
  • Injury Prediction in Competitive Runners Using Machine Learning
  • Protein Folding Using Machine Learning
  • Sentiment Analysis Using Machine Learning
  • Network Intrusion Detection System Using Machine Learning
  • Titanic- Machine Learning From Disaster
  • Adenovirus Disease Prediction for Child Healthcare Using Machine Learning
  • RNN for Sequence Labelling
  • CatBoost in Machine Learning
  • Cloud Computing Future Trends
  • Histogram of Oriented Gradients (HOG)
  • Implementation of neural network from scratch using NumPy
  • Introduction to SIFT( Scale Invariant Feature Transform)
  • Introduction to SURF (Speeded-Up Robust Features)
  • Kubernetes - load balancing service
  • Kubernetes Resource Model (KRM) and How to Make Use of YAML
  • Are Robots Self-Learning
  • Variational Autoencoders
  • What are the Security and Privacy Risks of VR and AR
  • What is a Large Language Model (LLM)
  • Privacy-preserving Machine Learning
  • Continual Learning in Machine Learning
  • Quantum Machine Learning (QML)
  • Split Single Column into Multiple Columns in PySpark DataFrame
  • Why should we use AutoML
  • Evaluation Metrics for Object Detection and Recognition
  • Mean Intersection over Union (mIoU) for image segmentation
  • YOLOV5-Object-Tracker-In-Videos
  • Predicting Salaries with Machine Learning
  • Fine-tuning Large Language Models
  • AutoML Workflow
  • Build Chatbot Webapp with LangChain
  • Building a Machine Learning Classification Model with PyCaret
  • Continuous Bag of Words (CBOW) in NLP
  • Deploying Scrapy Spider on ScrapingHub
  • Dynamic Pricing Using Machine Learning
  • How to Improve Neural Networks by Using Complex Numbers
  • Introduction to Bayesian Deep Learning
  • LiDAR: Light Detection and Ranging for 3D Reconstruction
  • Meta-Learning in Machine Learning
  • Object Recognition in Medical Imaging
  • Region-level Evaluation Metrics for Image Segmentation
  • Sarcasm Detection Using Neural Networks
  • SARSA Reinforcement Learning
  • Single Shot MultiBox Detector (SSD) using Neural Networking Approach
  • Stepwise Predictive Analysis in Machine Learning
  • Vision Transformers vs. Convolutional Neural Networks
  • V-Net in Image Segmentation
  • Forest Cover Type Prediction Using Machine Learning
  • Ada Boost algorithm in Machine Learning
  • Continuous Value Prediction
  • Bayesian Regression
  • Least Angle Regression
  • Linear Models
  • DNN Machine Learning
  • Why do we need to learn Machine Learning
  • Roles in Machine Learning
  • Clustering Performance Evaluation
  • Spectral Co-clustering
  • 7 Best R Packages for Machine Learning
  • Calculate Kurtosis
  • Machine Learning for Data Analysis
  • What are the benefits of 5G Technology for the Internet of Things
  • What is the Role of Machine Learning in IoT
  • Human Activity Recognition Using Machine Learning
  • Components of GIS
  • Attention Mechanism
  • Backpropagation- Algorithm
  • VGGNet-16 Architecture
  • Independent Component Analysis
  • Nonnegative Matrix Factorization
  • Sparse Inverse Covariance
  • Accuracy, Precision, Recall or F1
  • L1 and L2 Regularization
  • Maximum Likelihood Estimation
  • Kernel Principal Component Analysis (KPCA)
  • Latent Semantic Analysis
  • Overview of outlier detection methods
  • Robust Covariance Estimation
  • Spectral Bi-Clustering
  • Drift in Machine Learning
  • Credit Card Fraud Detection Using Machine Learning
  • KL-Divergence
  • Transformers Architecture
  • Novelty Detection with Local Outlier Factor
  • Novelty Detection
  • Introduction to Bayesian Linear Regression
  • Firefly Algorithm
  • Keras: Attention and Seq2Seq
  • A Guide Towards a Successful Machine Learning Project
  • ACF and PCF
  • Bayesian Hyperparameter Optimization for Machine Learning
  • Random Forest Hyperparameter tuning in python
  • Simulated Annealing
  • Top Benefits of Machine Learning in FinTech
  • Weight Initialisation
  • Density Estimation
  • Overlay Network
  • Micro, Macro Weighted Averages of F1 Score
  • Assumptions of Linear Regression
  • Evaluation Metrics for Clustering Algorithms
  • Frog Leap Algorithm
  • Isolation Forest
  • McNemar Test
  • Stochastic Optimization
  • Geomagnetic Field Using Machine Learning
  • Image Generation Using Machine Learning
  • Confidence Intervals
  • Facebook Prophet
  • Understanding Optimization Algorithms in Machine Learning
  • What Are Probabilistic Models in Machine Learning
  • How to choose the best Linear Regression model
  • How to Remove Non-Stationarity From Time Series
  • AutoEncoders
  • Cat Classification Using Machine Learning
  • AIC and BIC
  • Inception Model
  • Architecture of Machine Learning
  • Business Intelligence Vs Machine Learning
  • Guide to Cluster Analysis: Applications, Best Practices
  • Linear Regression using Gradient Descent
  • Text Clustering with K-Means
  • The Significance and Applications of Covariance Matrix
  • Stationarity Tests in Time Series
  • Graph Machine Learning
  • Introduction to XGBoost Algorithm in Machine Learning
  • Bahdanau Attention
  • Greedy Layer Wise Pre-Training
  • OneVsRestClassifier
  • Best Program for Machine Learning
  • Deep Boltzmann machines (DBMs) in machine learning
  • Find Patterns in Data Using Machine Learning
  • Generalized Linear Models
  • How to Implement Gradient Descent Optimization from Scratch
  • Interpreting Correlation Coefficients
  • Image Captioning Using Machine Learning
  • fit() vs predict() vs fit_predict() in Python scikit-learn
  • CNN Filters
  • Shannon Entropy
  • Time Series -Exponential Smoothing
  • AUC ROC Curve in Machine Learning
  • Vector Norms in Machine Learning
  • Swarm Intelligence
  • L1 and L2 Regularization Methods in Machine Learning
  • ML Approaches for Time Series
  • MSE and Bias-Variance Decomposition
  • Simple Exponential Smoothing
  • How to Optimise Machine Learning Model
  • Multiclass logistic regression from scratch
  • Lightbm Multilabel Classification
  • Monte Carlo Methods
  • What is Inverse Reinforcement learning
  • Content-Based Recommender System
  • Context-Awareness Recommender System
  • Predicting Flights Using Machine Learning
  • NTLK Corpus
  • Traditional Feature Engineering Models
  • Concept Drift and Model Decay in Machine Learning
  • Hierarchical Reinforcement Learning
  • What is Feature Scaling and Why is it Important in Machine Learning
  • Difference between Statistical Model and Machine Learning
  • Introduction to Ranking Algorithms in Machine Learning
  • Multicollinearity: Causes, Effects and Detection
  • Bag of N-Grams Model
  • TF-IDF Model

Related Tutorials

  • Tensorflow Tutorial
  • PyTorch Tutorial
  • Data Science Tutorial
  • AI Tutorial
  • NLP Tutorial
  • Reinforcement Learning

Interview Questions

  • Machine learning Interview

The hypothesis is a common term in Machine Learning and data science projects. As we know, machine learning is one of the most powerful technologies across the world, which helps us to predict results based on past experiences. Moreover, data scientists and ML professionals conduct experiments that aim to solve a problem. These ML professionals and data scientists make an initial assumption for the solution of the problem.

This assumption in Machine learning is known as Hypothesis. In Machine Learning, at various times, Hypothesis and Model are used interchangeably. However, a Hypothesis is an assumption made by scientists, whereas a model is a mathematical representation that is used to test the hypothesis. In this topic, "Hypothesis in Machine Learning," we will discuss a few important concepts related to a hypothesis in machine learning and their importance. So, let's start with a quick introduction to Hypothesis.

It is just a guess based on some known facts but has not yet been proven. A good hypothesis is testable, which results in either true or false.

: Let's understand the hypothesis with a common example. Some scientist claims that ultraviolet (UV) light can damage the eyes then it may also cause blindness.

In this example, a scientist just claims that UV rays are harmful to the eyes, but we assume they may cause blindness. However, it may or may not be possible. Hence, these types of assumptions are called a hypothesis.

The hypothesis is one of the commonly used concepts of statistics in Machine Learning. It is specifically used in Supervised Machine learning, where an ML model learns a function that best maps the input to corresponding outputs with the help of an available dataset.

There are some common methods given to find out the possible hypothesis from the Hypothesis space, where hypothesis space is represented by and hypothesis by Th ese are defined as follows:

It is used by supervised machine learning algorithms to determine the best possible hypothesis to describe the target function or best maps input to output.

It is often constrained by choice of the framing of the problem, the choice of model, and the choice of model configuration.

. It is primarily based on data as well as bias and restrictions applied to data.

Hence hypothesis (h) can be concluded as a single hypothesis that maps input to proper output and can be evaluated as well as used to make predictions.

The hypothesis (h) can be formulated in machine learning as follows:

Where,

Y: Range

m: Slope of the line which divided test data or changes in y divided by change in x.

x: domain

c: intercept (constant)

: Let's understand the hypothesis (h) and hypothesis space (H) with a two-dimensional coordinate plane showing the distribution of data as follows:

Hypothesis space (H) is the composition of all legal best possible ways to divide the coordinate plane so that it best maps input to proper output.

Further, each individual best possible way is called a hypothesis (h). Hence, the hypothesis and hypothesis space would be like this:

Similar to the hypothesis in machine learning, it is also considered an assumption of the output. However, it is falsifiable, which means it can be failed in the presence of sufficient evidence.

Unlike machine learning, we cannot accept any hypothesis in statistics because it is just an imaginary result and based on probability. Before start working on an experiment, we must be aware of two important types of hypotheses as follows:

A null hypothesis is a type of statistical hypothesis which tells that there is no statistically significant effect exists in the given set of observations. It is also known as conjecture and is used in quantitative analysis to test theories about markets, investment, and finance to decide whether an idea is true or false. An alternative hypothesis is a direct contradiction of the null hypothesis, which means if one of the two hypotheses is true, then the other must be false. In other words, an alternative hypothesis is a type of statistical hypothesis which tells that there is some significant effect that exists in the given set of observations.

The significance level is the primary thing that must be set before starting an experiment. It is useful to define the tolerance of error and the level at which effect can be considered significantly. During the testing process in an experiment, a 95% significance level is accepted, and the remaining 5% can be neglected. The significance level also tells the critical or threshold value. For e.g., in an experiment, if the significance level is set to 98%, then the critical value is 0.02%.

The p-value in statistics is defined as the evidence against a null hypothesis. In other words, P-value is the probability that a random chance generated the data or something else that is equal or rarer under the null hypothesis condition.

If the p-value is smaller, the evidence will be stronger, and vice-versa which means the null hypothesis can be rejected in testing. It is always represented in a decimal form, such as 0.035.

Whenever a statistical test is carried out on the population and sample to find out P-value, then it always depends upon the critical value. If the p-value is less than the critical value, then it shows the effect is significant, and the null hypothesis can be rejected. Further, if it is higher than the critical value, it shows that there is no significant effect and hence fails to reject the Null Hypothesis.

In the series of mapping instances of inputs to outputs in supervised machine learning, the hypothesis is a very useful concept that helps to approximate a target function in machine learning. It is available in all analytics domains and is also considered one of the important factors to check whether a change should be introduced or not. It covers the entire training data sets to efficiency as well as the performance of the models.

Hence, in this topic, we have covered various important concepts related to the hypothesis in machine learning and statistics and some important parameters such as p-value, significance level, etc., to understand hypothesis concepts in a better way.





Latest Courses

Python

We provides tutorials and interview questions of all technology like java tutorial, android, java frameworks

Contact info

G-13, 2nd Floor, Sec-3, Noida, UP, 201301, India

[email protected] .

Facebook

Online Compiler

Programmathically

Introduction to the hypothesis space and the bias-variance tradeoff in machine learning.

what is hypothesis space in machine learning

In this post, we introduce the hypothesis space and discuss how machine learning models function as hypotheses. Furthermore, we discuss the challenges encountered when choosing an appropriate machine learning hypothesis and building a model, such as overfitting, underfitting, and the bias-variance tradeoff.

The hypothesis space in machine learning is a set of all possible models that can be used to explain a data distribution given the limitations of that space. A linear hypothesis space is limited to the set of all linear models. If the data distribution follows a non-linear distribution, the linear hypothesis space might not contain a model that is appropriate for our needs.

To understand the concept of a hypothesis space, we need to learn to think of machine learning models as hypotheses.

The Machine Learning Model as Hypothesis

Generally speaking, a hypothesis is a potential explanation for an outcome or a phenomenon. In scientific inquiry, we test hypotheses to figure out how well and if at all they explain an outcome. In supervised machine learning, we are concerned with finding a function that maps from inputs to outputs.

But machine learning is inherently probabilistic. It is the art and science of deriving useful hypotheses from limited or incomplete data. Our functions are not axioms that explain the data perfectly, and for most real-life problems, we will never have all the data that exists. Accordingly, we will not find the one true function that perfectly describes the data. Instead, we find a function through training a model to map from known training input to known training output. This way, the model gradually approximates the assumed true function that describes the distribution of the data. So we treat our model as a hypothesis that needs to be tested as to how well it explains the output from a given input. We do this using a test or validation data set.

The Hypothesis Space

During the training process, we select a model from a hypothesis space that is subject to our constraints. For example, a linear hypothesis space only provides linear models. We can approximate data that follows a quadratic distribution using a model from the linear hypothesis space.

model from a linear hypothesis space

Of course, a linear model will never have the same predictive performance as a quadratic model, so we can adjust our hypothesis space to also include non-linear models or at least quadratic models.

model from a quadratic hypothesis space

The Data Generating Process

The data generating process describes a hypothetical process subject to some assumptions that make training a machine learning model possible. We need to assume that the data points are from the same distribution but are independent of each other. When these requirements are met, we say that the data is independent and identically distributed (i.i.d.).

Independent and Identically Distributed Data

How can we assume that a model trained on a training set will perform better than random guessing on new and previously unseen data? First of all, the training data needs to come from the same or at least a similar problem domain. If you want your model to predict stock prices, you need to train the model on stock price data or data that is similarly distributed. It wouldn’t make much sense to train it on whether data. Statistically, this means the data is identically distributed . But if data comes from the same problem, training data and test data might not be completely independent. To account for this, we need to make sure that the test data is not in any way influenced by the training data or vice versa. If you use a subset of the training data as your test set, the test data evidently is not independent of the training data. Statistically, we say the data must be independently distributed .

Overfitting and Underfitting

We want to select a model from the hypothesis space that explains the data sufficiently well. During training, we can make a model so complex that it perfectly fits every data point in the training dataset. But ultimately, the model should be able to predict outputs on previously unseen input data. The ability to do well when predicting outputs on previously unseen data is also known as generalization. There is an inherent conflict between those two requirements.

If we make the model so complex that it fits every point in the training data, it will pick up lots of noise and random variation specific to the training set, which might obscure the larger underlying patterns. As a result, it will be more sensitive to random fluctuations in new data and predict values that are far off. A model with this problem is said to overfit the training data and, as a result, to suffer from high variance .

a model that overfits the data

To avoid the problem of overfitting, we can choose a simpler model or use regularization techniques to prevent the model from fitting the training data too closely. The model should then be less influenced by random fluctuations and instead, focus on the larger underlying patterns in the data. The patterns are expected to be found in any dataset that comes from the same distribution. As a consequence, the model should generalize better on previously unseen data.

a model that underfits the data

But if we go too far, the model might become too simple or too constrained by regularization to accurately capture the patterns in the data. Then the model will neither generalize well nor fit the training data well. A model that exhibits this problem is said to underfit the data and to suffer from high bias . If the model is too simple to accurately capture the patterns in the data (for example, when using a linear model to fit non-linear data), its capacity is insufficient for the task at hand.

When training neural networks, for example, we go through multiple iterations of training in which the model learns to fit an increasingly complex function to the data. Typically, your training error will decrease during learning the more complex your model becomes and the better it learns to fit the data. In the beginning, the training error decreases rapidly. In later training iterations, it typically flattens out as it approaches the minimum possible error. Your test or generalization error should initially decrease as well, albeit likely at a slower pace than the training error. As long as the generalization error is decreasing, your model is underfitting because it doesn’t live up to its full capacity. After a number of training iterations, the generalization error will likely reach a trough and start to increase again. Once it starts to increase, your model is overfitting, and it is time to stop training.

overfitting vs underfitting

Ideally, you should stop training once your model reaches the lowest point of the generalization error. The gap between the minimum generalization error and no error at all is an irreducible error term known as the Bayes error that we won’t be able to completely get rid of in a probabilistic setting. But if the error term seems too large, you might be able to reduce it further by collecting more data, manipulating your model’s hyperparameters, or altogether picking a different model.

Bias Variance Tradeoff

We’ve talked about bias and variance in the previous section. Now it is time to clarify what we actually mean by these terms.

Understanding Bias and Variance

In a nutshell, bias measures if there is any systematic deviation from the correct value in a specific direction. If we could repeat the same process of constructing a model several times over, and the results predicted by our model always deviate in a certain direction, we would call the result biased.

Variance measures how much the results vary between model predictions. If you repeat the modeling process several times over and the results are scattered all across the board, the model exhibits high variance.

In their book “Noise” Daniel Kahnemann and his co-authors provide an intuitive example that helps understand the concept of bias and variance. Imagine you have four teams at the shooting range.

bias and variance

Team B is biased because the shots of its team members all deviate in a certain direction from the center. Team B also exhibits low variance because the shots of all the team members are relatively concentrated in one location. Team C has the opposite problem. The shots are scattered across the target with no discernible bias in a certain direction. Team D is both biased and has high variance. Team A would be the equivalent of a good model. The shots are in the center with little bias in one direction and little variance between the team members.

Generally speaking, linear models such as linear regression exhibit high bias and low variance. Nonlinear algorithms such as decision trees are more prone to overfitting the training data and thus exhibit high variance and low bias.

A linear model used with non-linear data would exhibit a bias to predict data points along a straight line instead of accomodating the curves. But they are not as susceptible to random fluctuations in the data. A nonlinear algorithm that is trained on noisy data with lots of deviations would be more capable of avoiding bias but more prone to incorporate the noise into its predictions. As a result, a small deviation in the test data might lead to very different predictions.

To get our model to learn the patterns in data, we need to reduce the training error while at the same time reducing the gap between the training and the testing error. In other words, we want to reduce both bias and variance. To a certain extent, we can reduce both by picking an appropriate model, collecting enough training data, selecting appropriate training features and hyperparameter values. At some point, we have to trade-off between minimizing bias and minimizing variance. How you balance this trade-off is up to you.

bias variance trade-off

The Bias Variance Decomposition

Mathematically, the total error can be decomposed into the bias and the variance according to the following formula.

Remember that Bayes’ error is an error that cannot be eliminated.

Our machine learning model represents an estimating function \hat f(X) for the true data generating function f(X) where X represents the predictors and y the output values.

Now the mean squared error of our model is the expected value of the squared difference of the output produced by the estimating function \hat f(X) and the true output Y.

The bias is a systematic deviation from the true value. We can measure it as the squared difference between the expected value produced by the estimating function (the model) and the values produced by the true data-generating function.

Of course, we don’t know the true data generating function, but we do know the observed outputs Y, which correspond to the values generated by f(x) plus an error term.

The variance of the model is the squared difference between the expected value and the actual values of the model.

Now that we have the bias and the variance, we can add them up along with the irreducible error to get the total error.

A machine learning model represents an approximation to the hypothesized function that generated the data. The chosen model is a hypothesis since we hypothesize that this model represents the true data generating function.

We choose the hypothesis from a hypothesis space that may be subject to certain constraints. For example, we can constrain the hypothesis space to the set of linear models.

When choosing a model, we aim to reduce the bias and the variance to prevent our model from either overfitting or underfitting the data. In the real world, we cannot completely eliminate bias and variance, and we have to trade-off between them. The total error produced by a model can be decomposed into the bias, the variance, and irreducible (Bayes) error.

what is hypothesis space in machine learning

About Author

what is hypothesis space in machine learning

Related Posts

backprop

Best Guesses: Understanding The Hypothesis in Machine Learning

Stewart Kaplan

  • February 22, 2024
  • General , Supervised Learning , Unsupervised Learning

Machine learning is a vast and complex field that has inherited many terms from other places all over the mathematical domain.

It can sometimes be challenging to get your head around all the different terminologies, never mind trying to understand how everything comes together.

In this blog post, we will focus on one particular concept: the hypothesis.

While you may think this is simple, there is a little caveat regarding machine learning.

The statistics side and the learning side.

Don’t worry; we’ll do a full breakdown below.

You’ll learn the following:

What Is a Hypothesis in Machine Learning?

  • Is This any different than the hypothesis in statistics?
  • What is the difference between the alternative hypothesis and the null?
  • Why do we restrict hypothesis space in artificial intelligence?
  • Example code performing hypothesis testing in machine learning

learning together

In machine learning, the term ‘hypothesis’ can refer to two things.

First, it can refer to the hypothesis space, the set of all possible training examples that could be used to predict or answer a new instance.

Second, it can refer to the traditional null and alternative hypotheses from statistics.

Since machine learning works so closely with statistics, 90% of the time, when someone is referencing the hypothesis, they’re referencing hypothesis tests from statistics.

Is This Any Different Than The Hypothesis In Statistics?

In statistics, the hypothesis is an assumption made about a population parameter.

The statistician’s goal is to prove it true or disprove it.

prove them wrong

This will take the form of two different hypotheses, one called the null, and one called the alternative.

Usually, you’ll establish your null hypothesis as an assumption that it equals some value.

For example, in Welch’s T-Test Of Unequal Variance, our null hypothesis is that the two means we are testing (population parameter) are equal.

This means our null hypothesis is that the two population means are the same.

We run our statistical tests, and if our p-value is significant (very low), we reject the null hypothesis.

This would mean that their population means are unequal for the two samples you are testing.

Usually, statisticians will use the significance level of .05 (a 5% risk of being wrong) when deciding what to use as the p-value cut-off.

What Is The Difference Between The Alternative Hypothesis And The Null?

The null hypothesis is our default assumption, which we are trying to prove correct.

The alternate hypothesis is usually the opposite of our null and is much broader in scope.

For most statistical tests, the null and alternative hypotheses are already defined.

You are then just trying to find “significant” evidence we can use to reject our null hypothesis.

can you prove it

These two hypotheses are easy to spot by their specific notation. The null hypothesis is usually denoted by H₀, while H₁ denotes the alternative hypothesis.

Example Code Performing Hypothesis Testing In Machine Learning

Since there are many different hypothesis tests in machine learning and data science, we will focus on one of my favorites.

This test is Welch’s T-Test Of Unequal Variance, where we are trying to determine if the population means of these two samples are different.

There are a couple of assumptions for this test, but we will ignore those for now and show the code.

You can read more about this here in our other post, Welch’s T-Test of Unequal Variance .

We see that our p-value is very low, and we reject the null hypothesis.

welch t test result with p-value

What Is The Difference Between The Biased And Unbiased Hypothesis Spaces?

The difference between the Biased and Unbiased hypothesis space is the number of possible training examples your algorithm has to predict.

The unbiased space has all of them, and the biased space only has the training examples you’ve supplied.

Since neither of these is optimal (one is too small, one is much too big), your algorithm creates generalized rules (inductive learning) to be able to handle examples it hasn’t seen before.

Here’s an example of each:

Example of The Biased Hypothesis Space In Machine Learning

The Biased Hypothesis space in machine learning is a biased subspace where your algorithm does not consider all training examples to make predictions.

This is easiest to see with an example.

Let’s say you have the following data:

Happy  and  Sunny  and  Stomach Full  = True

Whenever your algorithm sees those three together in the biased hypothesis space, it’ll automatically default to true.

This means when your algorithm sees:

Sad  and  Sunny  And  Stomach Full  = False

It’ll automatically default to False since it didn’t appear in our subspace.

This is a greedy approach, but it has some practical applications.

greedy

Example of the Unbiased Hypothesis Space In Machine Learning

The unbiased hypothesis space is a space where all combinations are stored.

We can use re-use our example above:

This would start to breakdown as

Happy  = True

Happy  and  Sunny  = True

Happy  and  Stomach Full  = True

Let’s say you have four options for each of the three choices.

This would mean our subspace would need 2^12 instances (4096) just for our little three-word problem.

This is practically impossible; the space would become huge.

subspace

So while it would be highly accurate, this has no scalability.

More reading on this idea can be found in our post, Inductive Bias In Machine Learning .

Why Do We Restrict Hypothesis Space In Artificial Intelligence?

We have to restrict the hypothesis space in machine learning. Without any restrictions, our domain becomes much too large, and we lose any form of scalability.

This is why our algorithm creates rules to handle examples that are seen in production. 

This gives our algorithms a generalized approach that will be able to handle all new examples that are in the same format.

Other Quick Machine Learning Tutorials

At EML, we have a ton of cool data science tutorials that break things down so anyone can understand them.

Below we’ve listed a few that are similar to this guide:

  • Instance-Based Learning in Machine Learning
  • Types of Data For Machine Learning
  • Verbose in Machine Learning
  • Generalization In Machine Learning
  • Epoch In Machine Learning
  • Inductive Bias in Machine Learning
  • Understanding The Hypothesis In Machine Learning
  • Zip Codes In Machine Learning
  • get_dummies() in Machine Learning
  • Bootstrapping In Machine Learning
  • X and Y in Machine Learning
  • F1 Score in Machine Learning
  • Recent Posts

Stewart Kaplan

  • How to Normalise Data [Boost Your Data Management Skills] - September 17, 2024
  • Are Companies Laying Off Software Engineers? Learn How to Secure Your Job [Must-Read Tips] - September 17, 2024
  • How much does a software development Intern at Workiva make? [Find Out Salary Details Here] - September 17, 2024

Hypothesis in Machine Learning: Comprehensive Overview(2021)

img

Introduction

Supervised machine learning (ML) is regularly portrayed as the issue of approximating an objective capacity that maps inputs to outputs. This portrayal is described as looking through and assessing competitor hypothesis from hypothesis spaces. 

The conversation of hypothesis in machine learning can be confused for a novice, particularly when “hypothesis” has a discrete, but correlated significance in statistics and all the more comprehensively in science.

Hypothesis Space (H)

The hypothesis space utilized by an ML system is the arrangement of all hypotheses that may be returned by it. It is ordinarily characterized by a Hypothesis Language, conceivably related to a Language Bias. 

Many ML algorithms depend on some sort of search methodology: given a set of perceptions and a space of all potential hypotheses that may be thought in the hypothesis space. They see in this space for those hypotheses that adequately furnish the data or are ideal concerning some other quality standard.

ML can be portrayed as the need to utilize accessible data objects to discover a function that most reliable maps inputs to output, alluded to as function estimate, where we surmised an anonymous objective function that can most reliably map inputs to outputs on all expected perceptions from the difficult domain. An illustration of a model that approximates the performs mappings and target function of inputs to outputs is known as hypothesis testing in machine learning.

The hypothesis in machine learning of all potential hypothesis that you are looking over, paying little mind to their structure. For the wellbeing of accommodation, the hypothesis class is normally compelled to be just each sort of function or model in turn, since learning techniques regularly just work on each type at a time. This doesn’t need to be the situation, however:

  • Hypothesis classes don’t need to comprise just one kind of function. If you’re looking through exponential, quadratic, and overall linear functions, those are what your joined hypothesis class contains.
  • Hypothesis classes additionally don’t need to comprise of just straightforward functions. If you figure out how to look over all piecewise-tanh2 functions, those functions are what your hypothesis class incorporates.

The enormous trade-off is that the bigger your hypothesis class in   machine learning, the better the best hypothesis models the basic genuine function, yet the harder it is to locate that best hypothesis. This is identified with the bias-variance trade-off.

  • Hypothesis (h)

A hypothesis function in machine learning is best describes the target. The hypothesis that an algorithm would concoct relies on the data and relies on the bias and restrictions that we have forced on the data.

The hypothesis formula in machine learning:

  • y  is range
  • m  changes in y divided by change in x
  • x  is domain
  • b  is intercept

The purpose of restricting hypothesis space in machine learning is so that these can fit well with the general data that is needed by the user. It checks the reality or deception of observations or inputs and examinations them appropriately. Subsequently, it is extremely helpful and it plays out the valuable function of mapping all the inputs till they come out as outputs. Consequently, the target functions are deliberately examined and restricted dependent on the outcomes (regardless of whether they are free of bias), in ML.

The hypothesis in machine learning space and inductive bias in machine learning is that the hypothesis space is a collection of valid Hypothesis, for example, every single desirable function, on the opposite side the inductive bias (otherwise called learning bias) of a learning algorithm is the series of expectations that the learner uses to foresee outputs of given sources of inputs that it has not experienced. Regression and Classification are a kind of realizing which relies upon continuous-valued and discrete-valued sequentially. This sort of issues (learnings) is called inductive learning issues since we distinguish a function by inducting it on data.

In the Maximum a Posteriori or MAP hypothesis in machine learning, enhancement gives a Bayesian probability structure to fitting model parameters to training data and another option and sibling may be a more normal Maximum Likelihood Estimation system. MAP learning chooses a solitary in all probability theory given the data. The hypothesis in machine learning earlier is as yet utilized and the technique is regularly more manageable than full Bayesian learning. 

Bayesian techniques can be utilized to decide the most plausible hypothesis in machine learning given the data the MAP hypothesis. This is the ideal hypothesis as no other hypothesis is more probable.

Hypothesis in machine learning or ML the applicant model that approximates a target function for mapping instances of inputs to outputs.

Hypothesis in statistics probabilistic clarification about the presence of a connection between observations. 

Hypothesis in science is a temporary clarification that fits the proof and can be disproved or confirmed. We can see that a hypothesis in machine learning draws upon the meaning of the hypothesis all the more extensively in science.

There are no right or wrong ways of learning AI and ML technologies – the more, the better! These valuable resources can be the starting point for your journey on how to learn Artificial Intelligence and Machine Learning. Do pursuing AI and ML interest you? If you want to step into the world of emerging tech, you can accelerate your career with this  Machine Learning And AI Courses   by Jigsaw Academy.

  • XGBoost Algorithm: An Easy Overview For 2021

tag-img

Fill in the details to know more

facebook

PEOPLE ALSO READ

what is hypothesis space in machine learning

Related Articles

what is hypothesis space in machine learning

From The Eyes Of Emerging Technologies: IPL Through The Ages

April 29, 2023

 width=

Personalized Teaching with AI: Revolutionizing Traditional Teaching Methods

April 28, 2023

img

Metaverse: The Virtual Universe and its impact on the World of Finance

April 13, 2023

img

Artificial Intelligence – Learning To Manage The Mind Created By The Human Mind!

March 22, 2023

what is hypothesis space in machine learning

Wake Up to the Importance of Sleep: Celebrating World Sleep Day!

March 18, 2023

what is hypothesis space in machine learning

Operations Management and AI: How Do They Work?

March 15, 2023

img

How Does BYOP(Bring Your Own Project) Help In Building Your Portfolio?

what is hypothesis space in machine learning

What Are the Ethics in Artificial Intelligence (AI)?

November 25, 2022

epoch in machine learning

What is Epoch in Machine Learning?| UNext

November 24, 2022

what is hypothesis space in machine learning

The Impact Of Artificial Intelligence (AI) in Cloud Computing

November 18, 2022

what is hypothesis space in machine learning

Role of Artificial Intelligence and Machine Learning in Supply Chain Management 

November 11, 2022

what is hypothesis space in machine learning

Best Python Libraries for Machine Learning in 2022

November 7, 2022

share

Are you ready to build your own career?

arrow

Query? Ask Us

what is hypothesis space in machine learning

Get In Touch

Experiential upskilling programs tailored to your organization's needs.

Add your details:

By proceeding, you agree to our privacy policy and also agree to receive information from UNext through WhatsApp & other means of communication.

& other means of communication. --> By proceeding, you agree to our privacy policy and also agree to receive information from UNext through WhatsApp & other means of communication.

Subscribe To Our Newsletter

Upgrade your inbox with our curated newletters once every month. We appreciate your support and will make sure to keep your subscription worthwhile

Enter Your Details ×

LEARN STATISTICS EASILY

LEARN STATISTICS EASILY

Learn Data Analysis Now!

LEARN STATISTICS EASILY LOGO 2

What is: Hypothesis Space

What is hypothesis space.

The term “hypothesis space” refers to the set of all possible hypotheses that can be formulated to explain a given set of data within the context of statistical modeling, machine learning, and data science. In essence, it encompasses every potential model or function that can be used to make predictions or inferences based on the available data. The hypothesis space is crucial in determining the effectiveness of a learning algorithm, as it defines the boundaries within which the algorithm operates. A well-defined hypothesis space allows for better generalization, enabling the model to perform effectively on unseen data.

 width=

Ad description. Lorem ipsum dolor sit amet, consectetur adipiscing elit.

Components of Hypothesis Space

A hypothesis space is typically composed of various models, each representing a different assumption about the underlying data-generating process. These models can range from simple linear functions to complex non-linear algorithms, such as neural networks. The complexity and richness of the hypothesis space are influenced by several factors, including the choice of features, the type of model employed, and the regularization techniques applied. For instance, a linear regression model has a relatively simple hypothesis space, while a deep learning model can possess a vast and intricate hypothesis space due to its multiple layers and parameters.

Importance of Hypothesis Space in Machine Learning

In machine learning, the hypothesis space plays a pivotal role in the learning process. It directly impacts the model’s ability to learn from data and make accurate predictions. A larger hypothesis space may provide the flexibility needed to capture complex patterns in the data, but it also increases the risk of overfitting, where the model learns noise rather than the underlying distribution. Conversely, a smaller hypothesis space may lead to underfitting, where the model fails to capture essential patterns. Therefore, finding the right balance in the hypothesis space is critical for achieving optimal model performance.

Exploration of Hypothesis Space

Exploring the hypothesis space involves evaluating different models and their performance on the training data. Techniques such as cross-validation are commonly employed to assess how well a model generalizes to unseen data. By partitioning the data into training and validation sets, data scientists can iteratively test various hypotheses and refine their models. This exploration is essential for identifying the most suitable hypothesis that balances complexity and accuracy, ultimately leading to better predictive performance.

Hypothesis Space and Regularization

Regularization techniques are often employed to manage the complexity of the hypothesis space. These techniques, such as L1 (Lasso) and L2 (Ridge) regularization, add a penalty term to the loss function, discouraging overly complex models. By constraining the hypothesis space, regularization helps prevent overfitting, ensuring that the model remains generalizable. This is particularly important in high-dimensional datasets where the risk of overfitting is heightened due to the increased number of features relative to the number of observations.

Dimensionality Reduction and Hypothesis Space

Dimensionality reduction techniques, such as Principal Component Analysis (PCA) and t-Distributed Stochastic Neighbor Embedding (t-SNE), can also influence the hypothesis space. By reducing the number of features, these techniques simplify the hypothesis space, making it easier for models to learn from the data. This simplification can lead to improved model performance, especially in cases where the original feature set contains redundant or irrelevant information. Consequently, dimensionality reduction serves as a valuable tool in the data preprocessing phase, enhancing the overall efficiency of the learning process.

Evaluating Hypothesis Space with Metrics

To assess the effectiveness of different hypotheses within the hypothesis space, various evaluation metrics are employed. Common metrics include accuracy, precision, recall, F1-score, and area under the ROC curve (AUC-ROC). These metrics provide insights into how well a model performs in terms of classification or regression tasks. By systematically evaluating different hypotheses against these metrics, data scientists can identify the most promising models and refine their approaches accordingly, ensuring that the selected hypothesis aligns with the desired outcomes.

Bayesian Perspective on Hypothesis Space

From a Bayesian perspective, the hypothesis space is treated probabilistically. Each hypothesis is assigned a prior probability, reflecting the belief in its validity before observing the data. As data is observed, these prior probabilities are updated to posterior probabilities using Bayes’ theorem. This approach allows for a more nuanced exploration of the hypothesis space, as it incorporates uncertainty and provides a framework for model comparison. Bayesian methods can be particularly useful in scenarios where prior knowledge is available, guiding the selection of hypotheses based on both empirical evidence and theoretical considerations.

Practical Applications of Hypothesis Space

In practical applications, understanding the hypothesis space is essential for various domains, including finance, healthcare, and marketing. For instance, in finance, different models may be hypothesized to predict stock prices based on historical data. In healthcare, hypothesis spaces can be constructed to identify risk factors for diseases based on patient data. In marketing, understanding customer behavior through various hypotheses can lead to more effective targeting strategies. By leveraging the concept of hypothesis space, practitioners can develop robust models that drive decision-making and enhance outcomes across diverse fields.

what is hypothesis space in machine learning

Stack Exchange Network

Stack Exchange network consists of 183 Q&A communities including Stack Overflow , the largest, most trusted online community for developers to learn, share their knowledge, and build their careers.

Q&A for work

Connect and share knowledge within a single location that is structured and easy to search.

What is the difference between hypothesis space and representational capacity?

I am reading Goodfellow et al Deeplearning Book . I found it difficult to understand the difference between the definition of the hypothesis space and representation capacity of a model.

In Chapter 5 , it is written about hypothesis space:

One way to control the capacity of a learning algorithm is by choosing its hypothesis space, the set of functions that the learning algorithm is allowed to select as being the solution.

And about representational capacity:

The model specifies which family of functions the learning algorithm can choose from when varying the parameters in order to reduce a training objective. This is called the representational capacity of the model.

If we take the linear regression model as an example and allow our output $y$ to takes polynomial inputs, I understand the hypothesis space as the ensemble of quadratic functions taking input $x$ , i.e $y = a_0 + a_1x + a_2x^2$ .

How is it different from the definition of the representational capacity, where parameters are $a_0$ , $a_1$ and $a_2$ ?

  • machine-learning
  • terminology
  • computational-learning-theory
  • hypothesis-class

nbro's user avatar

3 Answers 3

Consider a target function $f: x \mapsto f(x)$ .

A hypothesis refers to an approximation of $f$ . A hypothesis space refers to the set of possible approximations that an algorithm can create for $f$ . The hypothesis space consists of the set of functions the model is limited to learn. For instance, linear regression can be limited to linear functions as its hypothesis space, or it can be expanded to learn polynomials.

The representational capacity of a model determines the flexibility of it, its ability to fit a variety of functions (i.e. which functions the model is able to learn), at the same. It specifies the family of functions the learning algorithm can choose from.

Saurav Joshi's user avatar

  • 1 $\begingroup$ Does it mean that the set of functions described by the representational capacity is strictly included in the hypothesis space ? By definition, is it possible to have functions in the hypothesis space NOT described in the representational capacity ? $\endgroup$ –  Qwarzix Commented Aug 23, 2018 at 8:43
  • $\begingroup$ It's still pretty confusing to me. Most sources say that a "model" is an instance (after execution/training on data) of a "learning algorithm". How, then, can a model specify the family of functions the learning algorithm can choose from? It doesn't make sense to me. The authors of the book should've explained these concepts in more depth. $\endgroup$ –  Talendar Commented Oct 9, 2020 at 13:09

A hypothesis space is defined as the set of functions $\mathcal H$ that can be chosen by a learning algorithm to minimize loss (in general).

$$\mathcal H = \{h_1, h_2,....h_n\}$$

The hypothesis class can be finite or infinite, for example a discrete set of shapes to encircle certain portion of the input space is a finite hypothesis space, whereas hpyothesis space of parametrized functions like neural nets and linear regressors are infinite.

Although the term representational capacity is not in the vogue a rough definition woukd be: The representational capacity of a model, is the ability of its hypothesis space to approximate a complex function, with 0 error, which can only be approximated by infinitely many hypothesis spaces whose representational capacity is equal to or exceed the representational capacity required to approximate the complex function.

The most popular measure of representational capacity is the $\mathcal V$ $\mathcal C$ Dimension of a model. The upper bound for VC dimension ( $d$ ) of a model is: $$d \leq \log_2| \mathcal H|$$ where $|H|$ is the cardinality of the set of hypothesis space.

A hypothesis space/class is the set of functions that the learning algorithm considers when picking one function to minimize some risk/loss functional.

The capacity of a hypothesis space is a number or bound that quantifies the size (or richness) of the hypothesis space, i.e. the number (and type) of functions that can be represented by the hypothesis space. So a hypothesis space has a capacity. The two most famous measures of capacity are VC dimension and Rademacher complexity.

In other words, the hypothesis class is the object and the capacity is a property (that can be measured or quantified) of this object, but there is not a big difference between hypothesis class and its capacity, in the sense that a hypothesis class naturally defines a capacity, but two (different) hypothesis classes could have the same capacity.

Note that representational capacity (not capacity , which is common!) is not a standard term in computational learning theory, while hypothesis space/class is commonly used. For example, this famous book on machine learning and learning theory uses the term hypothesis class in many places, but it never uses the term representational capacity .

Your book's definition of representational capacity is bad , in my opinion, if representational capacity is supposed to be a synonym for capacity , given that that definition also coincides with the definition of hypothesis class, so your confusion is understandable.

  • 1 $\begingroup$ I agree with you. The authors of the book should've explained these concepts in more depth. Most sources say that a "model" is an instance (after execution/training on data) of a "learning algorithm". How, then, can a model specify the family of functions the learning algorithm can choose from? Also, as you pointed out, the definition of the terms "hypothesis space" and "representational capacity" given by the authors are practically the same, although they use the terms as if they represent different concepts. $\endgroup$ –  Talendar Commented Oct 9, 2020 at 13:18

You must log in to answer this question.

Not the answer you're looking for browse other questions tagged machine-learning terminology computational-learning-theory hypothesis-class capacity ..

  • Featured on Meta
  • User activation: Learnings and opportunities
  • Join Stack Overflow’s CEO and me for the first Stack IRL Community Event in...

Hot Network Questions

  • Why is Germany looking to import workers from Kenya, specifically?
  • Cartoon Network (Pakistan or India) show around 2007 or 2009 featuring a brother and sister who fight but later become allies
  • Are There U.S. Laws or Presidential Actions That Cannot Be Overturned by Successive Presidents?
  • LTspice Pulse Help
  • What was the document that Paul and Chloe signed with Sabrina?
  • A thought experiment regarding elliptical orbits
  • In The Martian, what does Mitch mean when he is talking to Teddy and says that the space program is not bigger than one person?
  • How can I calculate derivative of eigenstates numerically?
  • What is an apologetic to confront Schellenberg's non-resistant divine hiddenness argument?
  • (How) is it possible to let portable communication devices detonate via software?
  • Get the size (height, width and S=height*width in em) of the displayed image and draw it
  • meaning of a sentence from Agatha Christie (Murder of Roger Ackroyd)
  • What can I do to limit damage to a ceiling below bathroom after faucet leak?
  • My team is not responsive to group messages and other group initiatives. What should be the appropriate solution?
  • how does the US justice system combat rights violations that happen when bad practices are given a new name to avoid old rulings?
  • Understanding “Your true airspeed will be the same, but your airspeed as opposed to the ground is much faster.” Bhutan Paro International Airport PBH
  • Can I use a Forward Transformer in a Flyback Converter Circuit?
  • crontab schedule on Alpine Linux runs on days it's not supposed to run on
  • Can the signing of a bill by an acting governor be overturned by the actual governor?
  • 〈ü〉 vs 〈ue〉 in German, particularly names
  • Grothendieck topoi as a constructive property
  • Was the total glaciation of the world, a.k.a. snowball earth, due to Bok space clouds?
  • Why is 'это' neuter in this expression?
  • '05 Scion tC, bought used. 145k miles , unknown if spark plugs were ever changed. Should I change 'em?

what is hypothesis space in machine learning

ID3 Algorithm and Hypothesis space in Decision Tree Learning

The collection of potential decision trees is the hypothesis space searched by ID3. ID3 searches this hypothesis space in a hill-climbing fashion, starting with the empty tree and moving on to increasingly detailed hypotheses in pursuit of a decision tree that properly classifies the training data.

In this blog, we’ll have a look at the Hypothesis space in Decision Trees and the ID3 Algorithm. 

ID3 Algorithm: 

The ID3 algorithm (Iterative Dichotomiser 3) is a classification technique that uses a greedy approach to create a decision tree by picking the optimal attribute that delivers the most Information Gain (IG) or the lowest Entropy (H).

What is Information Gain and Entropy?  

Information gain: .

The assessment of changes in entropy after segmenting a dataset based on a characteristic is known as information gain.

It establishes how much information a feature provides about a class.

We divided the node and built the decision tree based on the value of information gained.

The greatest information gain node/attribute is split first in a decision tree method, which always strives to maximize the value of information gain. 

The formula for Information Gain: 

Entropy is a metric for determining the degree of impurity in a particular property. It denotes the unpredictability of data. The following formula may be used to compute entropy:

S stands for “total number of samples.”

P(yes) denotes the likelihood of a yes answer.

P(no) denotes the likelihood of a negative outcome.

  • Calculate the dataset’s entropy.
  • For each feature/attribute.

Determine the entropy for each of the category values.

Calculate the feature’s information gain.

  • Find the feature that provides the most information.
  • Repeat it till we get the tree we want.

Characteristics of ID3: 

  • ID3 takes a greedy approach, which means it might become caught in local optimums and hence cannot guarantee an optimal result.
  • ID3 has the potential to overfit the training data (to avoid overfitting, smaller decision trees should be preferred over larger ones).
  • This method creates tiny trees most of the time, however, it does not always yield the shortest tree feasible.
  • On continuous data, ID3 is not easy to use (if the values of any given attribute are continuous, then there are many more places to split the data on this attribute, and searching for the best value to split by takes a lot of time).

Over Fitting:  

Good generalization is the desired property in our decision trees (and, indeed, in all classification problems), as we noted before. 

This implies we want the model fit on the labeled training data to generate predictions that are as accurate as they are on new, unseen observations.

Capabilities and Limitations of ID3:

  • In relation to the given characteristics, ID3’s hypothesis space for all decision trees is a full set of finite discrete-valued functions.
  • As it searches across the space of decision trees, ID3 keeps just one current hypothesis. This differs from the prior version space candidate Elimination approach, which keeps the set of all hypotheses compatible with the training instances provided.
  • ID3 loses the capabilities that come with explicitly describing all consistent hypotheses by identifying only one hypothesis. It is unable to establish how many different decision trees are compatible with the supplied training data.
  • One benefit of incorporating all of the instances’ statistical features (e.g., information gain) is that the final search is less vulnerable to faults in individual training examples.
  • By altering its termination criterion to allow hypotheses that inadequately match the training data, ID3 may simply be modified to handle noisy training data.
  • In its purest form, ID3 does not go backward in its search. It never goes back to evaluate a choice after it has chosen an attribute to test at a specific level in the tree. As a result, it is vulnerable to the standard dangers of hill-climbing search without backtracking, resulting in local optimum but not globally optimal solutions.
  • At each stage of the search, ID3 uses all training instances to make statistically based judgments on how to refine its current hypothesis. This is in contrast to approaches that make incremental judgments based on individual training instances (e.g., FIND-S or CANDIDATE-ELIMINATION ).

Hypothesis Space Search by ID3: 

  • ID3 climbs the hill of knowledge acquisition by searching the space of feasible decision trees.
  • It looks for all finite discrete-valued functions in the whole space. Every function is represented by at least one tree.
  • It only holds one theory (unlike Candidate-Elimination). It is unable to inform us how many more feasible options exist.
  • It’s possible to get stranded in local optima.
  • At each phase, all training examples are used. Errors have a lower impact on the outcome.

Stack Exchange Network

Stack Exchange network consists of 183 Q&A communities including Stack Overflow , the largest, most trusted online community for developers to learn, share their knowledge, and build their careers.

Q&A for work

Connect and share knowledge within a single location that is structured and easy to search.

How to calculate hypothesis space

I'm trying to calculate the size of the hypothesis space of a function F. This function takes $N$ binary inputs and outputs a single binary classification.

With $N$ binary inputs, then the size of the domain must be $2^N$ . Then, I would think that for each of these possible $2^N$ instances there must be two hypotheses (one for each output). This would make the total number of hypotheses equal to $2 \times (2^N)$ .

I have read from other sources that the correct number of hypotheses is actually $2^{(2^N)}$ . What is the mistake in my thinking?

  • machine-learning
  • combinatorics

Carl's user avatar

  • 1 $\begingroup$ Could you please explain how you obtain the value of $2\times(2^N)$? That number does not appear to follow from the information you gave. Perhaps a complete enumeration of the cases when $N=2$ would clarify things. $\endgroup$ –  whuber ♦ Commented Dec 25, 2015 at 4:08
  • $\begingroup$ My thinking was that each combination of the N binary inputs could yield a result of either true or false (a binary output). With two possible outputs for each of the 2^N possible function evaluations, I calculated there must be 2*(2^N) different hypotheses. I hope that explains my thinking better. $\endgroup$ –  Isaac Getto Commented Dec 25, 2015 at 4:12
  • $\begingroup$ Please revisit your calculation, because it is incorrect. Explicit consideration of the case $N=2$ may help clear this up. $\endgroup$ –  whuber ♦ Commented Dec 26, 2015 at 14:21

3 Answers 3

In general, whenever we have a function $f: \mathcal{D} \rightarrow \mathcal{C}$ , the function can be considered as an element of the set $\mathcal{C}^\mathcal{D}$ (called the function space ). The set of all possible functions with domain $\mathcal{D}$ and codomain $\mathcal{C}$ is the full function space $\mathcal{C}^\mathcal{D}$ . Each function in the space can be considered as a list of outputs for each of the inputs --- the list has $|\mathcal{D}|$ elements and each element takes on one of $|\mathcal{C}|$ possible outputs. Consequently, using a simple application of the multiplication principle of counting , we have:

$$\begin{align} \text{No. of possible functions with domain } \mathcal{D} \text{ and codomain } \mathcal{C} &= \underbrace{|\mathcal{C}| \times \cdots \times |\mathcal{C}|}_{|\mathcal{D}| \text{ times}} \\[12pt] &= |\mathcal{C}|^{|\mathcal{D}|}. \\[6pt] \end{align}$$

Now, you have already correctly determined that there are $2^n$ possible inputs in the domain of the function, so we have $\mathcal{D} = 2^n$ in the present case. For every possible input in the domain the function output takes on one of two binary values, so we have $|\mathcal{C}| = 2$ . Consequently, in this case we have:

$$\text{No. of possible functions with domain } \mathcal{D} \text{ and codomain } \mathcal{C} = |\mathcal{C}|^{|\mathcal{D}|} = 2^{2^n}. $$

Ben's user avatar

  • 1 $\begingroup$ Your answer requires a a knowledge of set theory and would be confusing to someone who would not start "counting" from zero. I am not familiar with using domain and codomain in the context of set theory, so I do not fully understand your explanation. It is no doubt correct, but accessibility may be an issue. $\endgroup$ –  Carl Commented Feb 20, 2021 at 1:51
  • $\begingroup$ That is true, but I think this question is inherently a question about function spaces, which are generally explained in terms of sets. In order for the OP to obtain a good knowledge of this issue, I think he will ultimately need to read some material on function spaces and the rules of counting sets. $\endgroup$ –  Ben Commented Feb 20, 2021 at 3:24
  • $\begingroup$ I agree, but other people read this as well, and not everyone, e.g., me, wants to learn about set language. There is nothing wrong with your answer, nor with mine, the only difference is jargon. I tried for accessibility, you tried for precision of set language, question of taste really. $\endgroup$ –  Carl Commented Feb 20, 2021 at 7:04
  • $\begingroup$ P.S. +1 for your answer. $\endgroup$ –  Carl Commented Feb 20, 2021 at 11:14
  • 1 $\begingroup$ I like the fact that you have given a non-set based answer (+1). One of the nice things about having multiple answers is that you get explanations pitched with different levels of assumed knowledge and rigour. $\endgroup$ –  Ben Commented Jul 3, 2022 at 1:56

Think of the output as being a lock (0 closed, 1 opened) that is potentially opened by keys. That is, there might be no combination that can open the lock, or as many as $2^n$ keys that can open it. If the lock can be opened by only one key, then counting in binary it is some number between $0000\dots0000$ and $1111\dots1111$ for a binary number of length $n$ , and there are $2^n$ of those. Next we ask how may combinations of two keys can open the lock and there are $\left(\begin{array}{c}2^n\\2\end{array}\right)$ of those.

In general, we are adding up combinations

$$\left(\begin{array}{c}2^n\\0\end{array}\right)+\left(\begin{array}{c}2^n\\1\end{array}\right)+\left(\begin{array}{c}2^n\\2\end{array}\right)+\dots+\left(\begin{array}{c}2^n\\2^n-1\end{array}\right)+\left(\begin{array}{c}2^n\\2^n\end{array}\right).$$

Finally, as order does not matter, we can use the binomial theorem (see e.g., here ) to get $${m \choose 0} + {m \choose 1} + {m \choose 2} + \dots + {m \choose m} = 2^m,$$ which substituting $m=2^n$ leads us to $2^{2^n}$ , which is the answer you read.

  • $\begingroup$ @Sycorax Like this answer better? $\endgroup$ –  Carl Commented Jan 6, 2021 at 7:31
  • $\begingroup$ @Ben Thanks for the edit, but I'm curious, why improve an answer, which implies that you follow what is being said to the point of wanting to say it better, and then not upvote it? $\endgroup$ –  Carl Commented Feb 19, 2021 at 9:13
  • $\begingroup$ Hi @Carl: Glad you liked the edit. I haven't upvoted because I'm still undecided on whether I like this answer. While I like the attempt to use an example, I'm not sure if the keys/locks analogy really makes function spaces easier or harder to understand. I've just upvoted a couple of your other answers in the meantime, while I think more about it. (Since my edit was purely on formatting and syntactical grounds, it does not really imply a like or dislike of the answer; I just wanted to make the formatting nicer.) $\endgroup$ –  Ben Commented Feb 19, 2021 at 13:08

To calculate the Hypothesis Space:

enter image description here

if we have the given image above we can then figure it out the following way.

  • Count the number of attributes or features. In this case, we have four features or (4).

Analyze or if given what are the values corresponding to each feature (e.g. binary, or many different inputs). In this particular case, we have binary values (0/1).

So for each of the 2^4 attributes, the outputs can take 0 or 1.

itsmrbeltre's user avatar

Not the answer you're looking for? Browse other questions tagged machine-learning combinatorics or ask your own question .

  • Featured on Meta
  • User activation: Learnings and opportunities
  • Join Stack Overflow’s CEO and me for the first Stack IRL Community Event in...

Hot Network Questions

  • 〈ü〉 vs 〈ue〉 in German, particularly names
  • Grid-based pathfinding for a lot of agents: how to implement "Tight-Following"?
  • When did St Peter receive the Keys of Heaven?
  • How to sum all the elements in a group in specific row with the package `listofitems`?
  • crontab schedule on Alpine Linux runs on days it's not supposed to run on
  • Cutting a curve through a thick timber without waste
  • My one-liner 'delete old files' command finds the right files but will not delete them
  • How to rectify a mistake in DS 160
  • Grothendieck topoi as a constructive property
  • (How) is it possible to let portable communication devices detonate via software?
  • Necessary and sufficient condition to at least two cubic polynomial roots have absolute value lower than 1
  • Coloring a function based on its monotonicity
  • Movie from the fifties where aliens look human but wear sunglasses to hide that they have no irises (color) in their eyes - only whites!
  • Use the lower of two voltages when one is active
  • Can I use a machine washing to clean neoprene wetsuits/socks/hoods/gloves if I use cold water, no spinning, no bleach and a gentle detergent?
  • My math professor is Chinese. Is it okay for me to speak Chinese to her in office hours?
  • Can this phrase "the Conservatives opposite" be regarded as apposition structure?
  • Counting the number of meetings
  • Why a relay frequently clicks when a battery is low?
  • What was the newest chess piece
  • Hungarian Immigration wrote a code on my passport
  • I didn't make it into a graduate program last year. How can I make a compelling case with an unchanged profile?
  • Emergency belt repair
  • What is an apologetic to confront Schellenberg's non-resistant divine hiddenness argument?

what is hypothesis space in machine learning

  • Trending Categories

Data Structure

  • Selected Reading
  • UPSC IAS Exams Notes
  • Developer's Best Practices
  • Questions and Answers
  • Effective Resume Writing
  • HR Interview Questions
  • Computer Glossary

What is hypothesis in Machine Learning?

The hypothesis is a word that is frequently used in Machine Learning and data science initiatives. As we all know, machine learning is one of the most powerful technologies in the world, allowing us to anticipate outcomes based on previous experiences. Moreover, data scientists and ML specialists undertake experiments with the goal of solving an issue. These ML experts and data scientists make an initial guess on how to solve the challenge.

What is a Hypothesis?

A hypothesis is a conjecture or proposed explanation that is based on insufficient facts or assumptions. It is only a conjecture based on certain known facts that have yet to be confirmed. A good hypothesis is tested and yields either true or erroneous outcomes.

Let's look at an example to better grasp the hypothesis. According to some scientists, ultraviolet (UV) light can harm the eyes and induce blindness.

In this case, a scientist just states that UV rays are hazardous to the eyes, but people presume they can lead to blindness. Yet, it is conceivable that it will not be achievable. As a result, these kinds of assumptions are referred to as hypotheses.

Defining Hypothesis in Machine Learning

In machine learning, a hypothesis is a mathematical function or model that converts input data into output predictions. The model's first belief or explanation is based on the facts supplied. The hypothesis is typically expressed as a collection of parameters characterizing the behavior of the model.

If we're building a model to predict the price of a property based on its size and location. The hypothesis function may look something like this −

$$\mathrm{h(x)\:=\:θ0\:+\:θ1\:*\:x1\:+\:θ2\:*\:x2}$$

The hypothesis function is h(x), its input data is x, the model's parameters are 0, 1, and 2, and the features are x1 and x2.

The machine learning model's purpose is to discover the optimal values for parameters 0 through 2 that minimize the difference between projected and actual output labels.

To put it another way, we're looking for the hypothesis function that best represents the underlying link between the input and output data.

Types of Hypotheses in Machine Learning

The next step is to build a hypothesis after identifying the problem and obtaining evidence. A hypothesis is an explanation or solution to a problem based on insufficient data. It acts as a springboard for further investigation and experimentation. A hypothesis is a machine learning function that converts inputs to outputs based on some assumptions. A good hypothesis contributes to the creation of an accurate and efficient machine-learning model. Several machine learning theories are as follows −

1. Null Hypothesis

A null hypothesis is a basic hypothesis that states that no link exists between the independent and dependent variables. In other words, it assumes the independent variable has no influence on the dependent variable. It is symbolized by the symbol H0. If the p-value falls outside the significance level, the null hypothesis is typically rejected (). If the null hypothesis is correct, the coefficient of determination is the probability of rejecting it. A null hypothesis is involved in test findings such as t-tests and ANOVA.

2. Alternative Hypothesis

An alternative hypothesis is a hypothesis that contradicts the null hypothesis. It assumes that there is a relationship between the independent and dependent variables. In other words, it assumes that there is an effect of the independent variable on the dependent variable. It is denoted by Ha. An alternative hypothesis is generally accepted if the p-value is less than the significance level (α). An alternative hypothesis is also known as a research hypothesis.

3. One-tailed Hypothesis

A one-tailed test is a type of significance test in which the region of rejection is located at one end of the sample distribution. It denotes that the estimated test parameter is more or less than the crucial value, implying that the alternative hypothesis rather than the null hypothesis should be accepted. It is most commonly used in the chi-square distribution, where all of the crucial areas, related to, are put in either of the two tails. Left-tailed or right-tailed one-tailed tests are both possible.

4. Two-tailed Hypothesis

The two-tailed test is a hypothesis test in which the region of rejection or critical area is on both ends of the normal distribution. It determines whether the sample tested falls within or outside a certain range of values, and an alternative hypothesis is accepted if the calculated value falls in either of the two tails of the probability distribution. α is bifurcated into two equal parts, and the estimated parameter is either above or below the assumed parameter, so extreme values work as evidence against the null hypothesis.

Overall, the hypothesis plays a critical role in the machine learning model. It provides a starting point for the model to make predictions and helps to guide the learning process. The accuracy of the hypothesis is evaluated using various metrics like mean squared error or accuracy.

The hypothesis is a mathematical function or model that converts input data into output predictions, typically expressed as a collection of parameters characterizing the behavior of the model. It is an explanation or solution to a problem based on insufficient data. A good hypothesis contributes to the creation of an accurate and efficient machine-learning model. A two-tailed hypothesis is used when there is no prior knowledge or theoretical basis to infer a certain direction of the link.

Premansh Sharma

  • Related Articles
  • What is Machine Learning?
  • What is momentum in Machine Learning?
  • What is Epoch in Machine Learning?
  • What is Standardization in Machine Learning
  • What is Q-learning with respect to reinforcement learning in Machine Learning?
  • What is Bayes Theorem in Machine Learning
  • What is field Mapping in Machine Learning?
  • What is Parameter Extraction in Machine Learning
  • What is Tpot AutoML in machine learning?
  • What is Projection Perspective in Machine Learning?
  • What is Grouped Convolution in Machine Learning?
  • What is a Neural Network in Machine Learning?
  • What is corporate fraud detection in machine learning?
  • What is Linear Algebra Application in Machine Learning
  • What is Continuous Kernel Convolution in machine learning?

Kickstart Your Career

Get certified by completing the course

Stack Exchange Network

Stack Exchange network consists of 183 Q&A communities including Stack Overflow , the largest, most trusted online community for developers to learn, share their knowledge, and build their careers.

Q&A for work

Connect and share knowledge within a single location that is structured and easy to search.

What is the hypothesis space of decision tree learning?

Could you please explain what the hypothesis space for decision tree learning look like?

And what is the cardinality of this space?

  • decision-trees

Said Savci's user avatar

As per Tom Mitchell's,

".....For example, consider the space of hypotheses that could in principle be output by the above checkers learner. This hypothesis space consists of all evaluation functions that can be represented by some choice of values for the weights wo through w6. The learner's task is thus to search through this vast space to locate the hypothesis that is most consistent with the available training examples ....."

Hence , Basically all possible combination of distinct trees makes the hypothesis space.

Lets say if you have chosen to represent your function to be a linear line then all possible linear lines which go through the data (given input, output) makes up your hypothesis space.

Each tree= Single hypothesis , that says this tree shall best fit my data and predict the correct results

therefore combination of all such possible tress= hypothesis space.

Here is the snippet of PPT from lecture

enter image description here

Your Answer

Sign up or log in, post as a guest.

Required, but never shown

By clicking “Post Your Answer”, you agree to our terms of service and acknowledge you have read our privacy policy .

Not the answer you're looking for? Browse other questions tagged decision-trees or ask your own question .

  • The Overflow Blog
  • The world’s largest open-source business has plans for enhancing LLMs
  • Looking under the hood at the tech stack that powers multimodal AI
  • Featured on Meta
  • Join Stack Overflow’s CEO and me for the first Stack IRL Community Event in...
  • User activation: Learnings and opportunities

Hot Network Questions

  • LTspice Pulse Help
  • Grid-based pathfinding for a lot of agents: how to implement "Tight-Following"?
  • A word like "science/scientific" that can be used for ALL academic fields?
  • Ellipsoid name from semi-major axis and flattening
  • Example of a forcing notion with finite-predecessor condition that does not add reals
  • Need help in tikzpicture
  • Would a scientific theory of everything be falsifiable?
  • What is the best way to protect from polymorphic viruses?
  • My math professor is Chinese. Is it okay for me to speak Chinese to her in office hours?
  • How to sum all the elements in a group in specific row with the package `listofitems`?
  • Apple IIgs to VGA adapter
  • Understanding “Your true airspeed will be the same, but your airspeed as opposed to the ground is much faster.” Bhutan Paro International Airport PBH
  • How to narrow hotel map down to Old Town of Cologne, Germany
  • Emergency belt repair
  • How to make a soundless world
  • What does the intercept mean in a model where the predictor is substance use at baseline and the outcome is substance use over time?
  • My team is not responsive to group messages and other group initiatives. What should be the appropriate solution?
  • Can I install a screw all the way through these threaded fork crown holes?
  • Get the size (height, width and S=height*width in em) of the displayed image and draw it
  • Necessary and sufficient condition to at least two cubic polynomial roots have absolute value lower than 1
  • What can I do to limit damage to a ceiling below bathroom after faucet leak?
  • Movie from the fifties where aliens look human but wear sunglasses to hide that they have no irises (color) in their eyes - only whites!
  • Why does fdisk create a 512B partition when I enter +256K?
  • What was the newest chess piece

what is hypothesis space in machine learning

Stack Exchange Network

Stack Exchange network consists of 183 Q&A communities including Stack Overflow , the largest, most trusted online community for developers to learn, share their knowledge, and build their careers.

Q&A for work

Connect and share knowledge within a single location that is structured and easy to search.

hypothesis space - linear and logistic regression

I am new to machine learning and I came across the term "hypothesis space". I am trying to grasp what is it and especially am interested in dimension of this "space." For example in the context of linear regression, trying to fit a linear polynomial to the data, would the dimension of the hypothesis space be $2$ ? What about in the context of logistic regression?

  • machine-learning

funmath's user avatar

  • $\begingroup$ How was the term used? $\endgroup$ –  Michael Hardy Commented Apr 29, 2020 at 4:53
  • $\begingroup$ One often speaks of a "parameter space". In the simplest logistic regression problems, one has $$ \operatorname{logit} \Pr(Y_i=1) = \alpha + \beta x_i $$ where $$\operatorname{logit} p = \log \frac p {1-p}$$ and $\Pr(Y_i\in\{0,1\}) = 1.$ Then the parameter space is the set of all possible values of the two parameters $\alpha,\beta.$ And one considers hypotheses concerning the values of these two parameters. $\endgroup$ –  Michael Hardy Commented Apr 29, 2020 at 4:56
  • $\begingroup$ @MichaelHardy I think hypothesis space has more to do with function space as opposed to parameter space. I am unsure though if both end up have the same dimension. $\endgroup$ –  funmath Commented Apr 29, 2020 at 16:24
  • $\begingroup$ As I said: How was the term used? $\endgroup$ –  Michael Hardy Commented Apr 29, 2020 at 16:30
  • $\begingroup$ @MichaelHardy A hypothesis space refers to the set of possible approximations that algorithm can create for f. The hypothesis space consists of the set of functions the model is limited to learn. For instance, linear regression can be limited to linear functions as its hypothesis space. $\endgroup$ –  funmath Commented Apr 29, 2020 at 16:40

In the simplest instances of logistic regression one has independent random variables $Y_1,\ldots,Y_n$ for which $$ \begin{cases} \operatorname{logit} \Pr(Y_i=1) = \phantom{+(}\alpha + \beta x_i \\[8pt] \operatorname{logit} \Pr(Y_i=0) = -(\alpha+\beta x_i) \end{cases} $$ where $$ \operatorname{logit} p = \log \frac p {1-p}, $$ and

  • $\{(x_i, Y_i) : i=1,\ldots,n\}$ are observed;
  • $\alpha,\beta$ are not observed and are to be estimated based on the above observed data;
  • As mentioned, $Y_i$ are random variables. On the other had $x_i$ are treated as constant, i.e. non-random, despite the fact that they may change if a new sample of $n$ observations is taken, the justification being that one is really interested in the conditional distribution of $Y$ given $x.$

Least squares is not the method used for estimating $\alpha$ and $\beta;$ maximum likelihood is, and the MLE is found by iteratively re-weighted least squares.

The function of most interest my be $$ p = \operatorname{logit}^{-1} (\alpha + \beta x) = \frac 1 {1 + e^{-(\alpha+\beta x)}}. $$ Every such function is completely determined by the values of $\alpha$ and $\beta.$ And in this case $\alpha$ and $\beta$ can be any real numbers at all.

Therefore the hypothesis space, if that is defined as the set of functions the model is limited to learn, is a $2$ -dimensional manifold homeopmorphic to the plane.

When the mapping from the parameter space to the hypothesis space is one-to-one and continuous, then the dimension of the hypothesis space is the same as the dimension of the parameter space. And "continuous" may be best defined in this context in such a way that it's always continuous, i.e. the mapping itself determines the topology on the hypothesis space.

Michael Hardy's user avatar

You must log in to answer this question.

Not the answer you're looking for browse other questions tagged machine-learning ..

  • Featured on Meta
  • Join Stack Overflow’s CEO and me for the first Stack IRL Community Event in...
  • User activation: Learnings and opportunities
  • 2024 Election Results: Congratulations to our new moderator!

Hot Network Questions

  • Bach's figured bass notation?
  • Can noun phrase have only one word?
  • How to react to a rejection based on a single one-line negative review?
  • meaning of a sentence from Agatha Christie (Murder of Roger Ackroyd)
  • On Putting Parentheses Around Item Numbers with Enumitem
  • CX and CZ commutation
  • Can turbo trainers be easily damaged when instaling a cassette?
  • Can I install a screw all the way through these threaded fork crown holes?
  • Cartoon Network (Pakistan or India) show around 2007 or 2009 featuring a brother and sister who fight but later become allies
  • Can I use a Forward Transformer in a Flyback Converter Circuit?
  • Ellipsoid name from semi-major axis and flattening
  • Plotting fuctions with domain markers at endpoints
  • What does "either" refer to in "We don't have to run to phone booths anymore, either"?
  • Why is Germany looking to import workers from Kenya, specifically?
  • Determining Entropy in PHP
  • Coloring a function based on its monotonicity
  • A certain solution for Sine-Gordon Equation
  • "00000000000000"
  • Help with understanding a rigid geometry proof
  • A function to convert numbers from scientific notation to plain decimal
  • Smallest prime q such that concatenation (p+q)"q is a prime
  • What was the newest chess piece
  • Class and macro with same name from different libraries
  • Need help in tikzpicture

what is hypothesis space in machine learning

  • Data Science
  • Data Analysis
  • Data Visualization
  • Machine Learning
  • Deep Learning
  • Computer Vision
  • Artificial Intelligence
  • AI ML DS Interview Series
  • AI ML DS Projects series
  • Data Engineering
  • Web Scrapping

What is Latent Space in Deep Learning?

In deep learning, latent space is a concept that refers to a transformed, often lower-dimensional representation of data, capturing its essential features and underlying patterns. This abstract space allows models to handle complex data more effectively by focusing on the most significant aspects of the data.

The article will delve into the intricacies of latent space, its significance, and its applications across various types of neural networks.

Table of Content

What is Latent Space?

Why latent space matters in deep learning, latent space in different types of neural networks, visualizing latent space, applications of latent space in deep learning.

Latent space refers to an abstract, often lower-dimensional representation of data that is generated by a neural network. This space is termed “latent” because it captures hidden, underlying features of the data that are not directly observable in the input space. Essentially, it represents the data in a more compact form, where similar data points are positioned closer together.

  • Dimensionality Reduction : Latent space enables the reduction of the input data’s dimensionality while retaining essential features. This compression makes it easier to handle and process complex data.
  • Feature Learning : By encoding data into a latent space, neural networks can learn meaningful features and patterns that are not immediately apparent in the raw data.
  • Generative Modeling : In generative models, latent space is used to sample new data points. This capability is crucial for tasks such as image synthesis and text generation.

1. Autoencoders

Autoencoders are neural networks designed to learn efficient encodings of data. They consist of two main components:The latent space in autoencoders is crucial because it contains a compressed version of the input data. By minimizing the reconstruction error, autoencoders learn to represent the data in a lower-dimensional space while preserving its essential characteristics.

  • Encoder : Maps the input data to the latent space.
  • Decoder : Reconstructs the data from the latent space representation.

2. Variational Autoencoders (VAEs)

VAEs extend the concept of autoencoders by introducing probabilistic elements into the latent space. Instead of learning a deterministic mapping, VAEs learn a distribution over the latent space. This allows them to generate new, diverse samples by sampling from this learned distribution.

Key components of VAEs include:

  • Encoder : Outputs parameters of a probability distribution (mean and variance) rather than a fixed latent vector.
  • Decoder : Reconstructs the data from samples drawn from the latent distribution.

3. Generative Adversarial Networks (GANs)

GANs use latent space to generate new data samples through a two-network setup:The latent space in GANs represents a range of possible data samples, and the generator learns to map points in this space to realistic data.

  • Generator : Takes a random vector from the latent space and generates data samples.
  • Discriminator : Distinguishes between real data samples and those generated by the generator.

4. Recurrent Neural Networks (RNNs) and Long Short-Term Memory Networks (LSTMs)

In sequence modeling tasks, RNNs and LSTMs use latent space to capture the temporal dependencies and hidden states of the data. The latent space in these models helps in summarizing past information and predicting future sequences.

  • Hidden States : Represent the latent space where information about previous time steps is encoded.
  • Cell States (LSTMs) : Capture long-term dependencies and are part of the latent space in LSTMs.

5. Transformers

Transformers, used extensively in natural language processing, also utilize latent space. The model represents each input token in a high-dimensional space and processes it through attention mechanisms.

  • Attention Mechanism : Allows the model to focus on different parts of the input data, effectively working with latent representations to understand context and relationships between tokens.

Visualizing latent space can provide insights into how a model represents data. Techniques such as t-SNE (t-Distributed Stochastic Neighbor Embedding) and PCA (Principal Component Analysis) are often used to project high-dimensional latent spaces into 2D or 3D for visualization.

  • t-SNE : Useful for preserving local structures and visualizing clusters in the latent space.
  • PCA : Helps in understanding the principal components and overall variance in the latent space.

Visualizing Latent Space using PCA

In this section, we are going to visualize latent space using PCA by following these steps:

  • Data Preprocessing : We normalize the MNIST dataset and flatten the images for the autoencoder.
  • Autoencoder Architecture : The autoencoder consists of an encoder that compresses the data into a latent space and a decoder that reconstructs the original data from the latent space.
  • Training : We train the autoencoder on the MNIST dataset, which helps the model learn meaningful representations in the latent space.
  • Dimensionality Reduction : We use PCA to reduce the latent space dimensions to 2D for visualization. Alternatively, you can use t-SNE for potentially better visual separation.
  • Plotting : We use Matplotlib to create scatter plots of the latent space representations, providing a visual understanding of how the model organizes the data.

latent-space-PCA

  • Data Compression : Latent space representations can be used for compressing data, making storage and transmission more efficient.
  • Anomaly Detection : By analyzing the latent space, models can identify anomalies or outliers that deviate significantly from the learned distribution.
  • Data Generation : Latent space allows for the generation of new data samples, which is valuable in creative fields such as art, music, and synthetic data creation.
  • Transfer Learning : Latent space representations learned from one domain can be transferred to other domains, facilitating knowledge transfer and improving model performance on new tasks.

Latent space is a fundamental concept in deep learning, enabling models to encode, compress, and generate data in a more efficient and meaningful way. Understanding and leveraging latent space is crucial for tasks ranging from data generation to feature extraction and anomaly detection. As deep learning continues to advance, the exploration of latent spaces will remain central to developing more sophisticated and powerful models.

Please Login to comment...

Similar reads.

  • AI-ML-DS With Python
  • How to Watch NFL Games Live Streams Free
  • OpenAI o1 AI Model Launched: Explore o1-Preview, o1-Mini, Pricing & Comparison
  • How to Merge Cells in Google Sheets: Step by Step Guide
  • How to Lock Cells in Google Sheets : Step by Step Guide
  • #geekstreak2024 – 21 Days POTD Challenge Powered By Deutsche Bank

Improve your Coding Skills with Practice

 alt=

What kind of Experience do you want to share?

IMAGES

  1. Hypothesis in Machine Learning

    what is hypothesis space in machine learning

  2. PPT

    what is hypothesis space in machine learning

  3. Hypothesis in Machine Learning

    what is hypothesis space in machine learning

  4. Hypothesis in Machine Learning

    what is hypothesis space in machine learning

  5. Machine Learning Terminologies for Beginners

    what is hypothesis space in machine learning

  6. Concept Learning Concept Space Hypothesis Space Distinct Hypothesis Space Machine Learning Mahesh

    what is hypothesis space in machine learning

VIDEO

  1. Hypothesis Representation Stanford University Coursera

  2. Concept of Hypothesis

  3. Riemannian Geometry

  4. Unit-1 Machine Learning

  5. Machine Learning class 6

  6. 05 Hypothesis, Hypothesis Space and Hypothesis Constraints in Learning System

COMMENTS

  1. Hypothesis in Machine Learning

    Learn the concept of a hypothesis in machine learning, how it works, and how it is evaluated. A hypothesis is a function that best describes the target, and the hypothesis space is the set of all possible legal hypotheses.

  2. What's a Hypothesis Space?

    A hypothesis space is the set of models that an algorithm can learn from any dataset. Learn how to choose the right hypothesis space for your data, and how expressivity and interpretability affect the performance and understanding of your models.

  3. What exactly is a hypothesis space in machine learning?

    This set of observations can be used by a machine learning (ML) algorithm to learn a function f that is able to predict a value y for any input from the input space. ... The function f has to be chosen from the hypothesis space. To get a better idea: The input space is in the above given example $2^4$, its the number of possible inputs.

  4. What is a Hypothesis in Machine Learning?

    A hypothesis in machine learning is a candidate model that approximates a target function for mapping inputs to outputs. Learn the difference between a hypothesis in science, in statistics, and in machine learning, and how they are used in supervised learning.

  5. Hypothesis in Machine Learning

    Learn what is hypothesis and hypothesis space in machine learning, and how they are used to find the best function to map inputs to outputs. Also, compare hypothesis in machine learning and statistics, and understand the concepts of null hypothesis, alternative hypothesis, significance level, and p-value.

  6. Machine Learning: The Basics

    A learning rate or step-size parameter used by gradient-based methods. h() A hypothesis map that reads in features x of a data point and delivers a prediction ^y= h(x) for its label y. H A hypothesis space or model used by a ML method. The hypothesis space consists of di erent hypothesis maps h: X!Ybetween which the ML method has to choose. 8

  7. Introduction to the Hypothesis Space and the Bias-Variance Tradeoff in

    The hypothesis space in machine learning is a set of all possible models that can be used to explain a data distribution given the limitations of that space. A linear hypothesis space is limited to the set of all linear models. If the data distribution follows a non-linear distribution, the linear hypothesis space might not contain a model that ...

  8. Best Guesses: Understanding The Hypothesis in Machine Learning

    In machine learning, the term 'hypothesis' can refer to two things. First, it can refer to the hypothesis space, the set of all possible training examples that could be used to predict or answer a new instance. Second, it can refer to the traditional null and alternative hypotheses from statistics. Since machine learning works so closely ...

  9. PDF Machine Learning

    Machine Learning 10-701, Fall 2015 VC Dimension and Model Complexity Eric Xing Lecture 16, November 3, 2015 ... hypothesis space H defined over instance space X is the size of the largest finite subset of X shattered by H. If arbitrarily large finite sets of X can be shattered by H, then VC(H) .

  10. Hypothesis in Machine Learning: Comprehensive Overview(2021)

    The hypothesis in machine learning space and inductive bias in machine learning is that the hypothesis space is a collection of valid Hypothesis, for example, every single desirable function, on the opposite side the inductive bias (otherwise called learning bias) of a learning algorithm is the series of expectations that the learner uses to ...

  11. What is: Hypothesis Space

    In machine learning, the hypothesis space plays a pivotal role in the learning process. It directly impacts the model's ability to learn from data and make accurate predictions. A larger hypothesis space may provide the flexibility needed to capture complex patterns in the data, but it also increases the risk of overfitting, where the model ...

  12. Machine Learning 1.1: Hypothesis Spaces

    This video introduces the concept of a hypothesis space which is a restricted set of predictor functions that can be computed and manipulated efficiently giv...

  13. PDF LECTURE 16: LEARNING THEORY

    CS446 Machine Learning Shattering a set of instances A set of instances S is shattered by the hypothesis space H if and only if for every dichotomy of S there is a hypothesis h in H that is consistent with this dichotomy. - dichotomy: partition instances in S into + and - - one dichotomy = label all instances in a subset P⊆ S as

  14. PDF CS534: Machine Learning

    Hypothesis space. The space of all hypotheses that can, in principle, be output by a particular learning algorithm. Version Space. The space of all hypotheses in the hypothesis space that have not yet been ruled out by a training example. Training Sample (or Training Set or Training Data): a set of N training examples drawn according to P(x,y).

  15. machine learning

    A hypothesis space/class is the set of functions that the learning algorithm considers when picking one function to minimize some risk/loss functional.. The capacity of a hypothesis space is a number or bound that quantifies the size (or richness) of the hypothesis space, i.e. the number (and type) of functions that can be represented by the hypothesis space.

  16. Version space learning

    The intermediate (thin) rectangles represent the hypotheses in the version space. Version space learning is a logical approach to machine learning, specifically binary classification. Version space learning algorithms search a predefined space of hypotheses, viewed as a set of logical sentences. Formally, the hypothesis space is a disjunction ...

  17. ID3 Algorithm and Hypothesis space in Decision Tree Learning

    Hypothesis Space Search by ID3: ID3 climbs the hill of knowledge acquisition by searching the space of feasible decision trees. It looks for all finite discrete-valued functions in the whole space. Every function is represented by at least one tree. It only holds one theory (unlike Candidate-Elimination).

  18. machine learning

    To calculate the Hypothesis Space: if we have the given image above we can then figure it out the following way. Count the number of attributes or features. In this case, we have four features or (4). Analyze or if given what are the values corresponding to each feature (e.g. binary, or many different inputs).

  19. What is Hypothesis in Machine Learning? How to Form a Hypothesis?

    The hypothesis is a crucial aspect of Machine Learning and Data Science. It is present in all the domains of analytics and is the deciding factor of whether a change should be introduced or not. Be it pharma, software, sales, etc. A Hypothesis covers the complete training dataset to check the performance of the models from the Hypothesis space.

  20. ML

    The candidate elimination algorithm incrementally builds the version space given a hypothesis space H and a set E of examples. The examples are added one by one; each example possibly shrinks the version space by removing the hypotheses that are inconsistent with the example. ... Reinforcement Learning is a type of Machine Learning paradigms in ...

  21. What is hypothesis in Machine Learning?

    In machine learning, a hypothesis is a mathematical function or model that converts input data into output predictions. The model's first belief or explanation is based on the facts supplied. The hypothesis is typically expressed as a collection of parameters characterizing the behavior of the model. If we're building a model to predict the ...

  22. What is the hypothesis space of decision tree learning?

    This hypothesis space consists of all evaluation functions that can be represented by some choice of values for the weights wo through w6. The learner's task is thus to search through this vast space to locate the hypothesis that is most consistent with the available training examples....." Hence , Basically all possible combination of distinct ...

  23. machine learning

    Therefore the hypothesis space, if that is defined as the set of functions the model is limited to learn, is a $2$-dimensional manifold homeopmorphic to the plane. When the mapping from the parameter space to the hypothesis space is one-to-one and continuous, then the dimension of the hypothesis space is the same as the dimension of the ...

  24. What is Latent Space in Deep Learning?

    In deep learning, latent space is a concept that refers to a transformed, often lower-dimensional representation of data, capturing its essential features and underlying patterns. This abstract space allows models to handle complex data more effectively by focusing on the most significant aspects of the data. The article will delve into the intricacies of latent space, its significance, and ...